LSAT and Law School Admissions Forum

Get expert LSAT preparation and law school admissions advice from PowerScore Test Preparation.

 Administrator
PowerScore Staff
  • PowerScore Staff
  • Posts: 8916
  • Joined: Feb 02, 2011
|
#38060
Complete Question Explanation
(The complete setup for this game can be found here: lsat/viewtopic.php?t=14958)

The correct answer choice is (A)

This is the first of two Rule Substitution questions on the December 2013 LSAT. The question stem requires you to replace the first rule of the game, which created the following relationship:

PT71_D13 LG Explanations_Game #1_Setup_Diagram 2.png
The rule itself is relatively straightforward. Without it, we are left with the following sequencing relationships:

  • ..... ..... ..... ..... K :longline: J :longline: H

    ..... ..... ..... ..... ..... L :longline: G
Our job is to substitute the first rule with a logically equivalent condition, which would have the exact same effect on the order in which the films are released. While the wording of the correct answer can be difficult-to-impossible to predict, the four incorrect answer choices will either present rules that were not part of the original rule set (also known as Additional Effects conditions), or else they will only partially constrain the variables in the rule being substituted (i.e. Partial Match conditions).

Since Additional Effects are easier to spot, look for answer choices that contain such conditions first. You should immediately notice, for instance, that answer choices (B) and (D) add conditions that are extraneous to our original rule set: K need not be released earlier than L, and F does not have to be released earlier than K. Both of these answer choices should quickly be eliminated.

Among the Partial Match answer choices, (E) is the easiest one to eliminate. While the rule we are asked to substitute requires either F or K to be released first, this inference—by itself—does not have the exact same effect on the order in which the films are released. For instance, if K were released first, the relationship between F, J, and L would remain completely uncertain. Because answer choice (E) can limit K without limiting F in any manner, it is incorrect.

Next, let’s take a look at answer choice (C). This rule is consistent with our original diagram, in which—as discussed above—F must be released either first or second. However, this rule by itself does not require F to be released before both J and L. If F were released second, for instance, it would be possible that L is released first, which would violate the rule being replaced. Therefore, answer choice (C) contains a Partial Match, and must be eliminated.

Answer choice (A): This is the correct answer choice. If only K can be released earlier than F, it follows that F must be released earlier than all other films, with the exception of K. This stipulation is consistent with the original diagram, because the rule we are asked to replace forced F into one of the first two slots (before J, H, L, and G). Answer choice (A) does not therefore contain an Additional Effects rule. Answer choice (A) is not a Partial Match either: if F is released earlier than all other films except for K, it logically follows that F must be released earlier than J and L. Thus, the effect is identical, and this is the correct answer choice.
You do not have the required permissions to view the files attached to this post.
 Jkjones3789
  • Posts: 89
  • Joined: Mar 12, 2014
|
#14833
Hello, So for 5 I chose E because in my main diagram the not laws I had under the 1st slot only allowed KorF to be first. I knew it was E or maybe A. I went with E. I know it has to produce a rule that perfectly replaces the original. So could you please tell me why my inference was incorrect.
 Robert Carroll
PowerScore Staff
  • PowerScore Staff
  • Posts: 1787
  • Joined: Dec 06, 2013
|
#14840
jk,

Substitution questions like this can be difficult because you must match the conditions of the original rules exactly - no more restriction and no less restriction. Here, you correctly identified that, under the original rules, K or F had to be first. However, answer choice (E), while including this limit on what can go first, does not allow you to infer that J and L must be after F.

Think about this: imagine we have all the rules except for the one that question 5 substitutes out. Add answer choice (E) in: then F or K must be first. But what if K is first? Under our original rules, if K is first, F must be second, because J and L must come after F, H must come after J, and G must come after L. But under our substituted rule set, nothing requires F to be second! J, L, or F could be second. So answer choice (E) did not include all the restrictions in our original rules.

Answer choice (A) does. We can rephrase it as saying "If F is not first, K is first, and F is second (because nothing BUT K can come before F)." This matches our original set of conditions.

Robert Carroll
 cardinal2017
  • Posts: 19
  • Joined: Oct 23, 2016
|
#34447
I'd like to elaborate on why (C) can't be an answer over (A) here.

I chose (C) at first because the rule in question (to be substituted) leads to F being earlier than four other items which are J, H, L, and G.

And (A) sounds similar to (C) in that both put F in either first or second spot.

However, if (C) becomes the answer (F on first or second spot), the rule does not set forth the relative order between F and J &L.

In that case, L might come before F, which violates the original rule in question.

Therefore, we should choose (A) instead of (C), which more properly restricts the order centering F.
 Emily Haney-Caron
PowerScore Staff
  • PowerScore Staff
  • Posts: 577
  • Joined: Jan 12, 2012
|
#34504
Hi cardinal2017,

Excellent reasoning, here! You're exactly right. :-D
User avatar
 TootyFrooty
  • Posts: 74
  • Joined: Oct 13, 2023
|
#104743
Administrator wrote: Mon Jan 20, 2014 12:00 am Complete Question Explanation
(The complete setup for this game can be found here: lsat/viewtopic.php?t=14958)

The correct answer choice is (A)

This is the first of two Rule Substitution questions on the December 2013 LSAT. The question stem requires you to replace the first rule of the game, which created the following relationship:

PT71_D13 LG Explanations_Game #1_Setup_Diagram 2.png

The rule itself is relatively straightforward. Without it, we are left with the following sequencing relationships:
  • ..... ..... ..... ..... K :longline: J :longline: H

    ..... ..... ..... ..... ..... L :longline: G
Our job is to substitute the first rule with a logically equivalent condition, which would have the exact same effect on the order in which the films are released. While the wording of the correct answer can be difficult-to-impossible to predict, the four incorrect answer choices will either present rules that were not part of the original rule set (also known as Additional Effects conditions), or else they will only partially constrain the variables in the rule being substituted (i.e. Partial Match conditions).

Since Additional Effects are easier to spot, look for answer choices that contain such conditions first. You should immediately notice, for instance, that answer choices (B) and (D) add conditions that are extraneous to our original rule set: K need not be released earlier than L, and F does not have to be released earlier than K. Both of these answer choices should quickly be eliminated.

Among the Partial Match answer choices, (E) is the easiest one to eliminate. While the rule we are asked to substitute requires either F or K to be released first, this inference—by itself—does not have the exact same effect on the order in which the films are released. For instance, if K were released first, the relationship between F, J, and L would remain completely uncertain. Because answer choice (E) can limit K without limiting F in any manner, it is incorrect.

Next, let’s take a look at answer choice (C). This rule is consistent with our original diagram, in which—as discussed above—F must be released either first or second. However, this rule by itself does not require F to be released before both J and L. If F were released second, for instance, it would be possible that L is released first, which would violate the rule being replaced. Therefore, answer choice (C) contains a Partial Match, and must be eliminated.

Answer choice (A): This is the correct answer choice. If only K can be released earlier than F, it follows that F must be released earlier than all other films, with the exception of K. This stipulation is consistent with the original diagram, because the rule we are asked to replace forced F into one of the first two slots (before J, H, L, and G). Answer choice (A) does not therefore contain an Additional Effects rule. Answer choice (A) is not a Partial Match either: if F is released earlier than all other films except for K, it logically follows that F must be released earlier than J and L. Thus, the effect is identical, and this is the correct answer choice.

I'm having a hard time seeing why answer a is correct. In fact all answers seem incorrect to me. If K is released before F then that still leaves us with KJH and LG, and who's to say F can be released last? What prevents it from doing so?
 Adam Tyson
PowerScore Staff
  • PowerScore Staff
  • Posts: 5153
  • Joined: Apr 14, 2011
|
#104763
Answer A isn't saying that K IS released before F, TootyFrooty. It's saying that nothing else can be released before F. In other words, either F is first, or else K is first and F is second. K is the only thing that's allowed to go before H, which means that F must therefore be before J, L, H, and G, which is exactly the situation that we started with.
User avatar
 TootyFrooty
  • Posts: 74
  • Joined: Oct 13, 2023
|
#104793
Adam Tyson wrote: Sun Jan 07, 2024 8:09 pm Answer A isn't saying that K IS released before F, TootyFrooty. It's saying that nothing else can be released before F. In other words, either F is first, or else K is first and F is second. K is the only thing that's allowed to go before H, which means that F must therefore be before J, L, H, and G, which is exactly the situation that we started with.

Thanks. I understand that when it says Only K can be released before F, that means K is the only variable that can be released before F but I'm sort of still struggling to understand how that gives the same effect of the rules?

So lets say K is released before F then we know J and H must come after, but where's the link for L?
Then lets say F is released first, then what comes after? Again there's no link to L?
User avatar
 Hanin Abu Amara
PowerScore Staff
  • PowerScore Staff
  • Posts: 60
  • Joined: Mar 29, 2023
|
#105319
Hi TootyFrooty

Because only K can be released earlier than F that would mean that F is before everyone else which includes J and L.

so either K is first followed by F then everyone else. Or F is first. In both instances F is before J and L.

Hope that helps.

Hanin

Get the most out of your LSAT Prep Plus subscription.

Analyze and track your performance with our Testing and Analytics Package.